LSAT and Law School Admissions Forum

Get expert LSAT preparation and law school admissions advice from PowerScore Test Preparation.

User avatar
 Dave Killoran
PowerScore Staff
  • PowerScore Staff
  • Posts: 5852
  • Joined: Mar 25, 2011
|
#82690
Complete Question Explanation
(The complete setup for this game can be found here: lsat/viewtopic.php?t=867)

The correct answer choice is (C).

If you are given the order of teams in any round, then in the previous round for the first and fifth positions there can be only two possible teams that occupied that position: for position 1, the teams in position 1 and 2; for position 5, the teams in position 4 and 5. Question #24 uses this information to great effect. In question #24, the order of teams after three rounds is:

G4-Q24-d1.png

In attempting to determine the order of teams after the second round, it is important to realize that teams can only move up or down one position at a time at most. Thus, at the end of the second round either J or R must have been in position 1. Unfortunately, every answer choice lists J or R in position 1. Let us try position 5. At the end of the second round either M or S must have been in position 5. Because only answer choice (C) lists M or S in position 5, answer choice (C) must be the correct answer.
 cspertus
  • Posts: 7
  • Joined: Nov 03, 2020
|
#82969
I am having trouble seeing an efficient approach to this question, especially since I wasn't sure whether to start with an even or odd round. After toying with it I see that if you start with 1) even: J and M winning: RJSML 2) odd: L wins: RJSLM 3) L and R win: RJLSM but I'm not sure how to arrive at that answer quickly. Thank you!
 Robert Carroll
PowerScore Staff
  • PowerScore Staff
  • Posts: 1787
  • Joined: Dec 06, 2013
|
#83285
cspertus,

I think the first approach I would take is seeing if any answers choices stand out as involving particularly large changes of position after a single round. Because of the way the game works, a team can change AT MOST by one position after each round. Of course, not every team will - higher-ranked teams that win, lower-ranked teams that lose, and teams that don't even play won't change position in a single round, so changing position isn't ever required. But if a team does change position after a round, it would be by one position, up or down.

If M is going to be 5th after the third round, it must either have already been 5th after the second round, or just got there by dropping one position. Answer choices (A), (D), and (E) have M better than 4th - it's not possible for M to have been better than 4th, then drop to 5th in a single round. So those answer choices can be readily eliminated. Answer choice (B) has R 5th, which couldn't have been the case after two rounds - R is 1st after the third round, according to the local condition. So R couldn't have moved that far in a single round. That leaves only answer choice (C).

Note that I could get rid of every answer by looking for answers that made teams move too far in a single round. If, after checking those facts, I still had more than one answer left, I would have to explore which types of rounds occurred first, second, and third, and see which of the remaining answers was consistent with one such succession of rounds. That's going to be time-consuming - what I expect is that I can eliminate at least some answers by looking at excessive movements of teams. If I can eliminate 4 answers that way, then that's all I have to do, as here.

Robert Carroll

Get the most out of your LSAT Prep Plus subscription.

Analyze and track your performance with our Testing and Analytics Package.